2017 AMC 8 Problems/Problem 4

Revision as of 15:03, 22 November 2017 by LearningMath (talk | contribs) (Created page with "==Problem 4== When <math>0.000315</math> is multiplied by <math>7,928,564</math> the product is closest to which of the following? <math>\textbf{(A) }210\qquad\textbf{(B) }24...")
(diff) ← Older revision | Latest revision (diff) | Newer revision → (diff)

Problem 4

When $0.000315$ is multiplied by $7,928,564$ the product is closest to which of the following?

$\textbf{(A) }210\qquad\textbf{(B) }240\qquad\textbf{(C) }2100\qquad\textbf{(D) }2400\qquad\textbf{(E) }24000$

Solution:

We can approximate $7,928,564$ to $8,000,000,$ and $0.000315$ to $0.0003.$ Multiplying the two yields $2400.$ This gives our answer to be $\text{D)}2400.$